Table of contents

Video Lecture (swedish)

Intro to equilibrium

Equilibrium part 2

Static Equilibrium (Statisk jämvikt)

Statics

F=0M=0\begin{array}{l} \sum \mathit{\mathbf{F} }=0\\ \sum \mathit{\mathbf{M} }=0 \end{array}

Solution strategy

  1. What is known and unknown?

  2. Free body diagram

  3. Insert coordinate system

  4. Identify a suitable point for a moment equation.

  5. ΣM=0\displaystyle \Sigma \mathit{\mathbf{M} }=0

  6. ΣF=0\displaystyle \Sigma \mathit{\mathbf{F} }=0

  7. Solve the equation system for the unknowns

  8. Analyze the results, plot, visualize

  9. Conclusion

Example 1

A sphere with a mass mm is resting against smooth surfaces according to the figure above. Determine the normal force at AA and BB.

Solution

Welcome to the swamp of angles (vinkelträsket)! Insert a Cartesian coordinate system i the center of the sphere and create a free body diagram for the sphere!

Since the surfaces are smooth they can only give rise to normal forces. We thus have contact forces at AA and BB, as well as a weight force which acts on the center of the sphere. Since all forces are directed through the center of the sphere, force equilibrium will be enough to solve for all unknowns.

  Fi=0\sum_{\;} {\mathit{\mathbf{F} } }_i =0
clear
syms F_A F_B m g

We can handle the swamp of angles in a systematic way. Define the angles from the xx-axis ([1,0]\left\lbrack 1,0\right\rbrack) positively counter-clockwise.

We can thus create a unit vector as a function of the angle θ\theta:

e(θ):=[cos(θ),sin(θ),0]T\mathit{\mathbf{e} }\left(\theta \right):=\left\lbrack \cos \left(\theta \right),\sin \left(\theta \right),0\right\rbrack^T

In Matlab, this can be done using an e.g., an anonymous function:

e = @(theta)[cosd(theta); sind(theta); 0];

Now we can get the direction of FA{\mathit{\mathbf{F} } }_A by starting at 180{180}^{\circ } and rotating "backwards" 75{75}^{\circ } and then again 90{90}^{\circ } to point in the direction we see in the figure.

FFA = F_A*e(180-75-90)
(8700286382685973FA9007199254740992291404338770025FA11258999068426240) \def\arraystretch{1.4} \left(\begin{array}{c} \frac{8700286382685973\,F_A }{9007199254740992}\\ \frac{291404338770025\,F_A }{1125899906842624}\\ 0 \end{array}\right)

Matlab sometimes cannot simplify expressions other than approximating it with a fraction. This is accurate but not readable, for this reason we can display the answer as a decimal number with an arbitrary precision, here we choose two significant numbers.

vpa(FFA,2)
(0.97FA0.26FA0) \left(\begin{array}{c} 0.97\,F_A \\ 0.26\,F_A \\ 0 \end{array}\right)

Then we have FB{\mathit{\mathbf{F} } }_B:

FFB = F_B*e(30+90)
(FB23FB20) \def\arraystretch{1.5} \left(\begin{array}{c} -\frac{F_B }{2}\\ \frac{\sqrt{3}\,F_B }{2}\\ 0 \end{array}\right)
vpa(FFB,2)
(0.5FB0.87FB0) \left(\begin{array}{c} -0.5\,F_B \\ 0.87\,F_B \\ 0 \end{array}\right)

The weight vector is simply

FG = [0;-m*g;0]
(0mg0) \left(\begin{array}{c} 0\\ -mg\\ 0 \end{array}\right)

Equilibrium stated that:

ekv = FFA+FFB+FG == 0;

The above expression is exact, but hard to read, so we display the decimal form of it using vpa and then we can analyze the equation system.

vpa(ekv,2)
(0.97FA0.5FB=0.00.26FA+0.87FB1.0gm=0.00.0=0.0) \left(\begin{array}{c} 0.97\,F_A -0.5\,F_B =0.0\\ 0.26\,F_A +0.87\,F_B -1.0\,g\,m=0.0\\ 0.0=0.0 \end{array}\right)

Now we can send the equation to the solver and solve for FBF_B and FAF_A.

[F_A,F_B] = solve(ekv,[F_A,F_B]);

We simplify the expression

F_A = vpa(simplify(F_A),2)
0.52gm 0.52\,g\,m
F_B = vpa(simplify(F_B),2)
1.0gm 1.0\,g\,m

FBF_B is simplified to mgmg!

The conclusion is that FBF_B takes the whole wight while FAF_A only takes about half of the spheres weight. This seems reasonable judging from the figure and FBD.

For a complete analysis we can rotate the whole system 30{30}^{\circ } clockwise such that FBF_B is pointing straight up and FAF_A thus is zero. Try it to confirm.

Example 2

Determine the wire tensions which carry the traffic light. The light has a mass of mmkg.

Solution

Insert a coordinate system and create a FBD.

We'll use the same approach as in example 2. Define the angles from the xx-axis ([1,0]\left\lbrack 1,0\right\rbrack) positively counter-clockwise.

clear
e = @(theta)[cosd(theta); sind(theta); 0];

Similarly to example 2, since all forces are acting through the Origin, a force equilibrium is enough to determine the unknowns.

syms FA FB mg
FFA = FA*e(180-30); vpa(FFA,2)
(0.87FA0.5FA0) \left(\begin{array}{c} -0.87\,\textrm{FA}\\ 0.5\,\textrm{FA}\\ 0 \end{array}\right)
FFB = FB*e(45); vpa(FFB,2)
(0.71FB0.71FB0) \left(\begin{array}{c} 0.71\,\textrm{FB}\\ 0.71\,\textrm{FB}\\ 0 \end{array}\right)
FG = [0;-mg;0];

ekv = FFA+FFB+FG==0; vpa(ekv,2)
(0.71FB0.87FA=0.00.5FA+0.71FB1.0mg=0.00.0=0.0) \left(\begin{array}{c} 0.71\,\textrm{FB}-0.87\,\textrm{FA}=0.0\\ 0.5\,\textrm{FA}+0.71\,\textrm{FB}-1.0\,\textrm{mg}=0.0\\ 0.0=0.0 \end{array}\right)
[FA, FB] = solve(ekv,[FA,FB]);
FA = simplify(FA)
mg(31) \textrm{mg}\,{\left(\sqrt{3}-1\right)}
FB = simplify(FB)
2mg(33)2 -\frac{\sqrt{2}\,\textrm{mg}\,{\left(\sqrt{3}-3\right)} }{2}
vpa(FA,2)
0.73mg 0.73\,\textrm{mg}
vpa(FB,2)
0.9mg 0.9\,\textrm{mg}

Conclusion: FBF_B is bigger than FAF_A because the point B is higher. If both angles had been 30{30}^{\circ } then the wire forces would have been the same and equal to m  gm\;g.

Example 3

Determine the wire tensions as a function of the distance dd. Plot both forces and analyze the results!

Solution

In contrast to the previous two academic examples, forces in real life are typically always going along a line of action and are defined from one point to another. In this example, we have forces along a line which is hung up between three points.

We start by inserting a coordinate system according to the FBD. Then positional vectors.

OA=[10,0,0]T,OB=[20,0,0]T,OR=[0,d,0]T\overrightarrow{\textrm{OA} } ={\left\lbrack -10,0,0\right\rbrack }^T ,\overrightarrow{\textrm{OB} } ={\left\lbrack 20,0,0\right\rbrack }^T ,\overrightarrow{\textrm{OR} } ={\left\lbrack 0,-d,0\right\rbrack }^T
clear
syms d positive

We create a variable dd and set it to be positive, this will simplify the expressions. See for yourself, try and experiment!

OA = [-10, 0, 0].'; OB = [20, 0, 0].'; OR = [0, -d, 0].';

Then wire forces which act from point RR toward points AA and BB.

syms v FA FB
RA = OA-OR;
RB = OB-OR;
FFA = FA*RA/norm(RA)
(10FAd2+100FAdd2+1000) \left(\begin{array}{c} -\frac{10\,\textrm{FA} }{\sqrt{d^2 +100} }\\ \frac{\textrm{FA}\,d}{\sqrt{d^2 +100} }\\ 0 \end{array}\right)
FFB = FB*RB/norm(RB)
(20FBd2+400FBdd2+4000) \left(\begin{array}{c} \frac{20\,\textrm{FB} }{\sqrt{d^2 +400} }\\ \frac{\textrm{FB}\,d}{\sqrt{d^2 +400} }\\ 0 \end{array}\right)
FG = [0; -100; 0];

ekv = [FFA+FFB+FG==0]
(20FBd2+40010FAd2+100=0FAdd2+100+FBdd2+400100=00=0) \left(\begin{array}{c} \frac{20\,\textrm{FB} }{\sqrt{d^2 +400} }-\frac{10\,\textrm{FA} }{\sqrt{d^2 +100} }=0\\ \frac{\textrm{FA}\,d}{\sqrt{d^2 +100} }+\frac{\textrm{FB}\,d}{\sqrt{d^2 +400} }-100=0\\ 0=0 \end{array}\right)
[FA, FB] = solve(ekv, [FA, FB])
FA =
200d2+1003d \frac{200\,\sqrt{d^2 +100} }{3\,d}
FB =
100d2+4003d \frac{100\,\sqrt{d^2 +400} }{3\,d}
figure; hold on
fplot(FA,[1,20],'Color','r','LineWidth',2,'DisplayName','F_A')
fplot(FB,[1,20],'Color','b','LineWidth',2,'DisplayName','F_B')
legend
ylim([0,200])

Here we can see that the wire forces are tending towards infinity as d0d\to 0. This is intuitive. Again, we can set the distances such that the weight hangs symmetrically and check the feasibility of our model.

We end with a limit as dd\to \infty.

vpa(limit([FA,FB],d,inf),3)
(66.733.3) \left(\begin{array}{cc} 66.7 & 33.3 \end{array}\right)

Conclusion: FA66.7NF_A \to 66\ldotp 7\mathrm{N} and FB33.3NF_B \to 33\ldotp 3\mathrm{N} with large values for dd.

Example 4

Determine the wire force and the reaction forces in the Origin if the homogeneous rod has a mass of mm kg and is free to rotate at OO.

Solution

Insert a coordinate system and positional vectors to all important points. Use the defined points to describe new points. We want to vary the position of BB along the xx-axis as well as the slope, θ\theta of the rod.

clear
syms theta b
e = @(theta)[cosd(theta),sind(theta),0].';
OA = 1.5*e(theta);
OB = [-b, 0, 0].';
OG = OA/2;

Now, the forces, start with the wire force.

syms F_A
AB = OB-OA;
FFA = F_A*AB/norm(AB)
(FA(b+3cos(πθ180)2)σ13FAsin(πθ180)2σ10)where    σ1=b+3cos(πθ180)22+9sin(πθ180)24 \begin{array}{l} \left(\begin{array}{c} -\frac{F_A \,{\left(b+\frac{3\,\cos \left(\frac{\pi \,\theta }{180}\right)}{2}\right)} }{\sigma_1 }\\ -\frac{3\,F_A \,\sin \left(\frac{\pi \,\theta }{180}\right)}{2\,\sigma_1 }\\ 0 \end{array}\right)\\ \mathrm{}\\ \textrm{where}\\ \mathrm{}\\ \;\;\sigma_1 =\sqrt{ {\left|b+\frac{3\,\cos \left(\frac{\pi \,\theta }{180}\right)}{2}\right|}^2 +\frac{9\,{\left|\sin \left(\frac{\pi \,\theta }{180}\right)\right|}^2 }{4} } \end{array}

Weight force

syms mg
FG = [0, -mg, 0].';

Reaction force in O.

syms R_x R_y
FO = [R_x, R_y, 0].';

Equilibrium

We have   F=0\sum_{\;} \mathit{\mathbf{F} }=0 and   MO=0\sum_{\;} M_O =0.

ekv = [FFA+FG+FO==0;
       cross(OA,FFA)+cross(OG,FG)==0]
(RxFAσ2σ1=0Rymg3FAσ32σ1=00=00=00=03FAσ3σ22σ19FAcos(πθ180)σ34σ13mgcos(πθ180)4=0)where    σ1=σ22+9σ324    σ2=b+3cos(πθ180)2    σ3=sin(πθ180) \begin{array}{l} \left(\begin{array}{c} R_x -\frac{F_A \,\sigma_2 }{\sigma_1 }=0\\ R_y -\textrm{mg}-\frac{3\,F_A \,\sigma_3 }{2\,\sigma_1 }=0\\ 0=0\\ 0=0\\ 0=0\\ \frac{3\,F_A \,\sigma_3 \,\sigma_2 }{2\,\sigma_1 }-\frac{9\,F_A \,\cos \left(\frac{\pi \,\theta }{180}\right)\,\sigma_3 }{4\,\sigma_1 }-\frac{3\,\textrm{mg}\,\cos \left(\frac{\pi \,\theta }{180}\right)}{4}=0 \end{array}\right)\\ \mathrm{}\\ \textrm{where}\\ \mathrm{}\\ \;\;\sigma_1 =\sqrt{ {\left|\sigma_2 \right|}^2 +\frac{9\,{\left|\sigma_3 \right|}^2 }{4} }\\ \mathrm{}\\ \;\;\sigma_2 =b+\frac{3\,\cos \left(\frac{\pi \,\theta }{180}\right)}{2}\\ \mathrm{}\\ \;\;\sigma_3 =\sin \left(\frac{\pi \,\theta }{180}\right) \end{array}

As we can see, we always get 6 equations and in 2D we always have three real equations, ΣFx=0,ΣFy=0,ΣMz=0\Sigma F_x =0,\Sigma F_y =0,\Sigma M_z =0.

We have three equations and three unknown variables: Rx,Ry  &  FAR_x ,R_y \;\&\;F_A

Solve the system of equations

[R_x, R_y, F_A] = solve(ekv,[R_x, R_y, F_A] )
R_x =
mgcos(πθ180)(2b+3cos(πθ180))4bsin(πθ180) \frac{\textrm{mg}\,\cos \left(\frac{\pi \,\theta }{180}\right)\,{\left(2\,b+3\,\cos \left(\frac{\pi \,\theta }{180}\right)\right)} }{4\,b\,\sin \left(\frac{\pi \,\theta }{180}\right)}
R_y =
mg(4b+3cos(πθ180))4b \frac{\textrm{mg}\,{\left(4\,b+3\,\cos \left(\frac{\pi \,\theta }{180}\right)\right)} }{4\,b}
F_A =
mgcos(πθ180)b+3cos(πθ180)22+9sin(πθ180)242bsin(πθ180) \frac{\textrm{mg}\,\cos \left(\frac{\pi \,\theta }{180}\right)\,\sqrt{ {\left|b+\frac{3\,\cos \left(\frac{\pi \,\theta }{180}\right)}{2}\right|}^2 +\frac{9\,{\left|\sin \left(\frac{\pi \,\theta }{180}\right)\right|}^2 }{4} } }{2\,b\,\sin \left(\frac{\pi \,\theta }{180}\right)}

Analysis

The resulting expressions are hard to make sense of! We need to plot!

We will set θ=60°\theta =60\degree and vary bb. Then we'll set b=1.5b=1\ldotp 5 and vary θ\theta.

Visualize stuff as much as possible, the eye is excellent at discovering interesting things!

mg = 1; theta = 60;
figure; hold on;
fplot(subs(F_A),[0.25,2],'Displayname','F_A(\theta=60^\circ)')
fplot(subs(R_x),[0.25,2],'Displayname','R_x(\theta=60^\circ)')
fplot(subs(R_y),[0.25,2],'Displayname','R_y(\theta=60^\circ)')
legend; xlabel('b'); ylabel('F/mg');

It makes sense that all forces are tending towards infinity as BB tends towards the Origin.

mg = 1; b = 1.5; syms theta
figure; hold on;
fplot(subs(F_A),[0, 90],'Displayname','F_A(b=1.5)')
fplot(subs(R_x),[0, 90],'Displayname','R_x(b=1.5)')
fplot(subs(R_y),[0, 90],'Displayname','R_y(b=1.5)')
legend; xlabel('\theta'); ylabel('F/mg');
axis([0,90,0,5])

ax = gca;
chart = ax.Children(1);
datatip(chart,90,1);

As θ\theta approached 90°90\degree the rod is vertical and all the weight is directly above the support, thus the weight vector and reaction vector cancel out, no force is acting in the horizontal direction and the wire force is thus zero. Makes perfect sense.

Example 5

A rectangular board with a mass of mmkg is carried in a horizontal position according to the figure above.

Create a FBD of the board, the people only exsert a force in the positive z-direction. Determine how much each person is lifting and which one of them is lifting in the least. Do you even lift bro?

Solution

Start the usual way, add a coordinate system and forces, create a FBD.

clear
OA = [0.360, 0, 0].'; OB = [0.720, 0.480+0.720, 0].';
OC = [0.720+1.680, 0.480, 0].'; OG = [0.720+1.680, 0.480+0.720,0].'/2;

The forces

syms FAz FBz FCz mg
FA = [0, 0, FAz].'; FB = [0,0,FBz].'; FC = [0,0,FCz].'; FG = [0,0,-mg].';

Equilibrium.

ΣF=FA+FB+FC+FG\Sigma \mathit{\mathbf{F} }={\mathit{\mathbf{F} } }_A +{\mathit{\mathbf{F} } }_B +{\mathit{\mathbf{F} } }_C +{\mathit{\mathbf{F} } }_G

Let's do a moment equation around the Origin.

ΣMO=OA×FA+OB×FB+OC×FC+OG×FG\Sigma M_O =\overrightarrow{\textrm{OA} } \times {\mathit{\mathbf{F} } }_A +\overrightarrow{\textrm{OB} } \times {\mathit{\mathbf{F} } }_B +\overrightarrow{\textrm{OC} } \times {\mathit{\mathbf{F} } }_C +\overrightarrow{\textrm{OG} } \times {\mathit{\mathbf{F} } }_G
ekv = [FA+FB+FC+FG;... 
       cross(OA,FA)+cross(OB,FB)+cross(OC,FC)+cross(OG,FG)]==0
(0=00=0FAz+FBz+FCzmg=06FBz5+12FCz253mg5=06mg518FBz2512FCz59FAz25=00=0) \left(\begin{array}{c} 0=0\\ 0=0\\ \textrm{FAz}+\textrm{FBz}+\textrm{FCz}-\textrm{mg}=0\\ \frac{6\,\textrm{FBz} }{5}+\frac{12\,\textrm{FCz} }{25}-\frac{3\,\textrm{mg} }{5}=0\\ \frac{6\,\textrm{mg} }{5}-\frac{18\,\textrm{FBz} }{25}-\frac{12\,\textrm{FCz} }{5}-\frac{9\,\textrm{FAz} }{25}=0\\ 0=0 \end{array}\right)
[FAz, FBz, FCz] = solve(ekv, [FAz, FBz, FCz]);
vpa([FAz, FBz, FCz],3)
(0.291mg0.361mg0.348mg) \left(\begin{array}{ccc} 0.291\,\textrm{mg} & 0.361\,\textrm{mg} & 0.348\,\textrm{mg} \end{array}\right)

Feasibility, sanity check

sum([FAz, FBz, FCz])
mg \textrm{mg}

Conclusion: The dude in A is lifting the least.

Example 6

A light crane as rigged according to the figure above. Determine, as a function of xx, the reaction forces in the xx-direction an the two wire forces. The crane is free to rotate about the Origin. The hook-carriage can travel along the x-axis.

Determine the minimum value of FO/FLF_O /F_L where FOF_O is the magnitude of the reaction force in OO and FLF_L is the magnitude of the load at the carriage.

Solution

ΣF=0\Sigma \mathit{\mathbf{F} }=0 ΣMO=0\Sigma M_O =0

FBD with a inserted coordinate system and forces.

clear
OA = [6,0,0].'; OB = [0, -2, 3].'; OC = [3, 0, 0].';
OD = [0, 4, 0].'; 
syms x real positive
OL = [x, 0, 0].';

Force vectors in OO, wires, etc.

syms F_L positive real
syms FA FC FOx FOy FOz
AB = OB-OA;
CD = OD-OC;
FFA = FA*AB/norm(AB)
(6FA72FA73FA7) \left(\begin{array}{c} -\frac{6\,\textrm{FA} }{7}\\ -\frac{2\,\textrm{FA} }{7}\\ \frac{3\,\textrm{FA} }{7} \end{array}\right)
FFC = FC*CD/norm(CD)
(3FC54FC50) \left(\begin{array}{c} -\frac{3\,\textrm{FC} }{5}\\ \frac{4\,\textrm{FC} }{5}\\ 0 \end{array}\right)
FFO = [FOx, FOy, FOz].';
FFL = [0, 0, -F_L].';

Set up the force and moment equations and solve for the unknowns.

ekv = [FFA+FFC+FFO+F_L;
       cross(OA,FFA)+cross(OL,FFL)+cross(OC,FFC)]==0
(FOx3FC56FA7+FL=04FC52FA7+FOy+FL=03FA7+FOz+FL=00=0FLx18FA7=012FC512FA7=0) \left(\begin{array}{c} \textrm{FOx}-\frac{3\,\textrm{FC} }{5}-\frac{6\,\textrm{FA} }{7}+F_L =0\\ \frac{4\,\textrm{FC} }{5}-\frac{2\,\textrm{FA} }{7}+\textrm{FOy}+F_L =0\\ \frac{3\,\textrm{FA} }{7}+\textrm{FOz}+F_L =0\\ 0=0\\ F_L \,x-\frac{18\,\textrm{FA} }{7}=0\\ \frac{12\,\textrm{FC} }{5}-\frac{12\,\textrm{FA} }{7}=0 \end{array}\right)
[FA, FC, FOx, FOy, FOz] = solve(ekv, [FA, FC, FOx, FOy, FOz])
FA =
7FLx18 \frac{7\,F_L \,x}{18}
FC =
5FLx18 \frac{5\,F_L \,x}{18}
FOx =
FLx2FL \frac{F_L \,x}{2}-F_L
FOy =
FLFLx9 -F_L -\frac{F_L \,x}{9}
FOz =
FLFLx6 -F_L -\frac{F_L \,x}{6}

Finally we compute FO/FLF_O /F_L. Studying a model of a moving load, e.g., a car that drives over a bridge, is very common for engineers. The picture of various loads as a function of the loads position is called the influence line diagram. Here is such a diagram

FO = simplify(subs(norm(FFO)))
2FL47x272x+48618 \frac{\sqrt{2}\,F_L \,\sqrt{47\,x^2 -72\,x+486} }{18}
kvot = simplify(FO/F_L)
247x272x+48618 \frac{\sqrt{2}\,\sqrt{47\,x^2 -72\,x+486} }{18}
figure
fplot(kvot, [0,2])
xlabel('x'); ylabel('F_O/F_L')

The minimum can be read from the graph or calculated with a bit of calculus:

x_min = vpa(solve(diff(kvot,x)==0,x),3)
0.766 0.766
FO_min = vpa(subs(kvot,x,x_min),3)
1.68 1.68

The resulting force in the xx-direction is given by

FOx
FLx2FL \frac{F_L \,x}{2}-F_L